page887      Permutations, cosets and Lagrange's theorem      Matthias Lorentzen...mattegrisenforlag.com


Look at the picture beneath, then scroll down to the question and click the correct Answer button.

Question

Is it true that H is a subgroup of `S_3`: `H < S_3` ?